0 Daumen
2,3k Aufrufe

Aufgabe:

z^2=-i

Lösungsmengen bestimmen
Problem/Ansatz:

Ich weiß zwar wie man die Lösungsmenge bestimmt, nur weiß ich nicht, wie ich den Winkel berechne, wenn der Realteil=0  ist : tan(alpha)=b/a=-1/0 !

Avatar von

2 Antworten

+1 Daumen

Wolframalpha ist ein Klick entfernt

https://www.wolframalpha.com/input/?i=-i

Das sind Winkel die sollte man auswendig wissen.

blob.png

Avatar von 479 k 🚀

Danke dir, nur darf ich Wolframalpha nicht in der Klausur verwenden.

Gefragt war ja auch nach dem Rechenweg und nicht nach der Instant Lösung.

Es ist klar das du es nicht in der Klausur verwenden darfst. Daher solltest du dich ja auch mit solchen Dingen vorher auseinandersetzen.

Zum Beispiel das es hier nicht geht den arctan() zu benutzen weil du in dem Fall durch 0 teilen müsstest.

Wie du rechentechnisch dort hin gelangst findest du auf Wikipedia

https://de.wikipedia.org/wiki/Komplexe_Zahl#Von_der_algebraischen_Form_in_die_Polarform

blob.png

0 Daumen

mach ne Skizze wo -i liegt, den Winkel kann man ablesen =270° .

Avatar von 37 k

Danke dir aber in der Klausur kann ich schlecht eine Skizze machen und außerdem würde ich gerne wissen wollen, wie man es rechnerisch macht.


Ich weiß, dass ich den Winkel -Pi/2 verwenden muss, da der Realteil 0 ist, nur komme ich so nicht auf das geforderte Ergebnis.

Laut meiner Rechnung wäre z1=-Pi/2 *1/2 =-Pi/4  und

z2= -Pi/2 *1/2 + 2*2Pi*1/2 = -Pi/4 + 2Pi = 7/8 Pi

Wenn du es nicht schaffts, in einer Klausur ein Kreuz zu malen und dann -i zu finden ist Hopfen und Malz verloren. Kann man auch auf einem Schmierblatt machen. Aber egal, rechnerisch geht es auch.

-pi/2 ist dasselbe wie 270° , da nur um 2pi verschoben.

tan(alpha)=-1/0"=" -∞

kannst du du auch auflösen,

arctan(-∞)=-pi/2

z2= cos(3/4Pi)+i*sin(3/4Pi) habe ich raus.

Wie komme ich bei z1 auf z2= cos(7/4Pi)+i*sin(7/4Pi) auf ?

Wo liegt der Fehler in meiner Rechnung zu z1 ?


Ps: Denk Winkel habe ich ja schon raus : "Ich weiß, dass ich den Winkel -Pi/2 verwenden muss, da der Realteil 0 ist..."

-i=e^{-pi/2}

z=±Sqrt(-i)=±e^{-pi/4}

=± ( cos(-pi/4)+i* sin(-pi/4))

=± (1/sqrt(2) -i 1/sqrt(2))


Deine Rechnung im ersten Kommentator kann ich nicht nachvollziehen, da fehlt überall  der Faktor i. In der Musterlösung schreibt man alles mit 7/4 pi, weil

-pi/4 +2pi=7/4 pi, die trigonometrischen Funktionen sind 2 pi periodisch.

Bei meiner Rechnung gehe ich nach der Formel :


1.)Koordinaten in Polarform

2.)Polarform in "e"-Form

3)Phi1=Phi(alt)/n     und   z2= Phi(alt)/n +2Pi/n   etc....


s. hier :

blob.png

Ein anderes Problem?

Stell deine Frage

Willkommen bei der Mathelounge! Stell deine Frage einfach und kostenlos

x
Made by a lovely community